PATHO 2 Exam 3

Ace your homework & exams now with Quizwiz!

A nurse who works in the office of an endocrinologist is orienting a new staff member. Which of the following teaching points is the nurse justified in including in the orientation? Select all that apply. A) "A bodily process can be the result of the combined effect of several different hormones from different sources." B) "A single hormone can act not only on one process or organ but often on several different locations or processes." C) "It's common for production of hormones to be far removed from the tissue where they ultimately exert their effect." D) "Sometimes hormones act locally on the area where they were produced, like in the case of paracrine and autocrine actions." E) "The regulation in homeostasis requires that hormones be absent from the body when their effect is not needed."

A) "A bodily process can be the result of the combined effect of several different hormones from different sources." B) "A single hormone can act not only on one process or organ but often on several different locations or processes." C) "It's common for production of hormones to be far removed from the tissue where they ultimately exert their effect." D) "Sometimes hormones act locally on the area where they were produced, like in the case of paracrine and autocrine actions." A single hormone can exert various effects in different tissues or, conversely, a single function can be regulated by several different hormones. Hormones act both distant from their source and more locally, as in the case of autocrine and paracrine actions. Hormones are normally present at all times.

Two nursing students are debating the merits and demerits of infant circumcision. Which of the following statements is most accurate? A) "Circumcised men tend to have a lower incidence of penile cancer." B) "Getting circumcised basically rules out the possibility of getting Peyronie disease later in life." C) "Circumcision reduces pressure on the deep dorsal vein and the dorsal artery, making erection easier later in life." D) "The odds of getting infant priapism fall with circumcision."

A) "Circumcised men tend to have a lower incidence of penile cancer." A correlation between circumcision and lower incidence of penile cancer has been noted. Circumcision is unlikely to affect the development of Peyronie disease or priapism, and is not noted to influence the ease of attaining or maintaining erection.

A 57-year-old woman has been diagnosed with atrophic vaginitis and has expressed surprise to her care provider, citing a lifetime largely free of gynecological health problems. She has asked what may have contributed to her problem. How can the care provider best respond? A) "The lower levels of estrogen since you've begun menopause make your vagina prone to infection." B) "Vaginitis is not usually the direct result of any single problem, but rather an inevitability of the vaginal dryness that accompanies menopause." C) "This type of vaginitis is most commonly a symptom of a latent sexually transmitted infection that you may have contracted in the distant past." D) "The exact cause of this problem isn't known, but it can usually be resolved with a diet high in probiotic bacteria."

A) "The lower levels of estrogen since you've begun menopause make your vagina prone to infection." The lack of vaginal epithelial regeneration after menopause predisposes older woman to vaginitis. It is not necessarily a result of vaginal dryness and is not likely sexually transmitted. The etiology is not unknown, and diet alone is unlikely to resolve the problem.

A 21-year-old college football player has acknowledged to a nurse at a campus clinic that he has been injecting androgens in an effort to increase the size of his musculature and enhance his on-field performance. Which of the following teaching points should the nurse include in a conversation with the player? A) "You run a real risk of developing acne and developing breasts by doing this." B) "You will indeed be able to increase your muscles, but they will rebound by shrinking smaller than they ever were within a few months." C) "Most of the people who do this cause themselves to be permanently sterile." D) "A reduction in the size of your penis and the production of mutated sperm can be an effect of steroid use."

A) "You run a real risk of developing acne and developing breasts by doing this." Testicular atrophy and gynecomastia are identified as the effects of anabolic steroid use. Rebound muscle atrophy is not a noted phenomenon that accompanies the steroid use, nor is a permanent cessation of spermatogenesis. Changes in penis size and mutation in sperm cell divisions are similarly unlikely.

A female patient presented to her primary care physician with classic signs and symptoms of Cushing syndrome. Upon testing, it was discovered that the patient had vaginal small cell carcinoma. How can the health care providers explain her Cushing syndrome signs and symptoms to this patient? A) "Your tumor in your vagina is secreting a hormone called adrenocorticotropic hormone (ACTH), which is responsible for these signs and symptoms." B) "We are going to have to run some more tests. We think you might have a problem with your pituitary gland." C) "There is no connection between the Cushing syndrome and the vaginal carcinoma. You have two very distinct problems occurring at the same time." D) "We need to check your thyroid. Your Cushing syndrome may be caused by hypofunction of this gland."

A) "Your tumor in your vagina is secreting a hormone called adrenocorticotropic hormone (ACTH), which is responsible for these signs and symptoms." Hyperfunction is usually associated with excessive hormone production. This can result from excessive stimulation and hyperplasia of the endocrine gland or from a hormone-producing tumor. A clinical example of this phenomenon is evidenced by the case of a woman with vaginal small cell carcinoma who also presented with Cushing syndrome. After testing, it was determined that the tumor is secreting ACTH. In this situation, the cause is not related to a pituitary problem. There is a connection between Cushing syndrome and the carcinoma. The thyroid gland is not responsible for Cushing syndrome.

Of the following patient conditions, which patients would be at risk for experiencing a thyroid problem due to a decrease in thyroxine-binding globulin (TBG)? Select all that apply. A) A 55-year-old male with cirrhosis due to alcohol abuse B) A 47-year-old female experiencing hot flashes and excess diaphoresis related to menopause C) A 75-year-old man receiving chronic glucocorticoid therapy to treat his severe chronic obstructive pulmonary disease (COPD) D) A 18-year-old female anorexia nervosa patient weighing 78 lb and has consumed no protein for the past 3 years

A) A 55-year-old male with cirrhosis due to alcohol abuse C) A 75-year-old man receiving chronic glucocorticoid therapy to treat his severe chronic obstructive pulmonary disease (COPD) D) A 18-year-old female anorexia nervosa patient weighing 78 lb and has consumed no protein for the past 3 years A number of disease conditions and pharmacologic agents can decrease the amount of binding protein in the plasma or influence the binding of hormone. Glucocorticoid medications and systemic disease conditions such as protein malnutrition, nephritic syndrome, and cirrhosis decrease TBG concentrations.

A 40-year-old male client with multiple health problems has been diagnosed with a testosterone deficiency. The nurse knows that which of the following assessment findings would correlate with a testosterone deficiency? Select all that apply. A) A high-pitched voice B) Long-lasting chronic hip dysplasia C) Complaints of multiple upper respiratory infections D) A low muscle mass in proportion to his total body weight E) Hot flashes and diaphoresis

A) A high-pitched voice D) A low muscle mass in proportion to his total body weight Vulnerability to infection and hip dysplasia would likely not be a manifestation of low testosterone. Low muscle mass, hot flashes and diaphoresis, and a lack of male voice characteristics could develop secondary to testosterone deficiency.

Of the following list of nursing interventions, which would be considered priority when managing a patient with life-threatening myxedematous coma? Select all that apply. A) Administer 3% sodium IV solution to increase sodium levels. B) Administer 50% dextrose to raise glucose levels. C) Place on oxygen therapy to encourage deep breathing. D) Place on a warming bed to raise body temperature. E) Administer sedatives frequently to prevent seizures.

A) Administer 3% sodium IV solution to increase sodium levels. B) Administer 50% dextrose to raise glucose levels. C) Place on oxygen therapy to encourage deep breathing. Myxedematous coma is a life-threatening, end-stage expression of hypothyroidism. It is characterized by coma, hypothermia, CV collapse, hypoventilation, and severe metabolic disorders that include low sodium, low glucose, and lactic acidosis. Treatment includes aggressive management of precipitating factors; supportive therapy such as management of CV status, hyponatremia, and hypoglycemia; and thyroid replacement therapy. If hypothermia is present, active rewarming is contraindicated because it may induce vasodilation and vascular collapse. Administering sedatives frequently could be harmful since the person is unable to metabolize sedatives, analgesics, and anesthetic drugs.

A nurse on a medical unit is providing care for a 37-year-old female patient who has a diagnosis of Graves disease. Which of the following treatments would the nurse most likely anticipate providing for the client? A) -Adrenergic-blocking medications to reduce sympathetic nervous stimulation B) Administration of levothyroxine to supplement thyroid function C) Calcium channel blocking medications to reduce heart rate and cardiac risks D) Administration of somatostatin analogs to inhibit GH production

A) B-Adrenergic-blocking medications to reduce sympathetic nervous stimulation The hyperthyroidism that constitutes Graves disease can often be mitigated by the administration of -adrenergic-blocking medications. Levothyroxine would be used to address hypothyroidism, and calcium channel blockers are not an identified treatment modality for Graves disease. Somatostatin analogs are used to treat GH excess.

A 31-year-old woman and her husband have presented to their family physician due to their inability to conceive a child after trying for the last 18 months. Which of the following will the nurse practitioner want to specifically rule out as potential contributing factors? Select all that apply. A) Cryptorchidism in the husband B) Slow maturation of the endothelial lining after ovulation C) Low levels of LH and FSH in the wife D) Large amounts of clear, stretchy cervical mucus E) Sexually transmitted diseases like gonorrhea or chlamydial infection.

A) Cryptorchidism in the husband B) Slow maturation of the endothelial lining after ovulation C) Low levels of LH and FSH in the wife E) Sexually transmitted diseases like gonorrhea or chlamydial infection. Cryptorchidism, luteal phase defects, low pituitary hormone levels, and an inability of the tubes to pick up an ovum can all contribute to infertility. A large amount of clear, stretchy cervical mucus is a normal finding. Cervical cultures for gonorrhea, chlamydial infection, and mycoplasmal infection should be obtained and treatment instituted as needed.

Long, stretchy cervical mucus that exhibits ferning on a microscope slide is characteristic of which of the following? A) Crystallization of inorganic salts B) Low estrogen levels C) High progesterone levels D) Low human chorionic gonadotropin levels

A) Crystallization of inorganic salts Cervical mucus that exhibits ferning and so-called spinnbarkeit occurs right around the time of ovulation (midcycle) due to increased water content and alteration in the concentration of inorganic salts. This is influenced by high serum levels of estrogen, which lead to the LH spike that promotes ovulation (bursting of the oocyte from the mature follicle). Progesterone levels increase only after ovulation during the luteal phase, at which point the cervical mucus "dries up" (becomes more scant).

Of the following list of medications, which would likely be prescribed to a patient with benign prostatic hyperplasia (BPH) as a way to decrease the prostate size by blocking the effects of androgens on the prostate? A) Finasteride (Proscar), a 5a-reductase inhibitor B) Imdur (isosorbide mononitrate), a vasodilator C) Birth control pills containing both estrogen and progestin D) Leuprolide (Lupron), a gonadotropin-releasing hormone analog

A) Finasteride (Proscar), a 5a-reductase inhibitor Finasteride (Proscar), a 5a-reductase inhibitor, reduces prostate size by blocking the effect of androgens on the prostate. Vasodilators, BCPs, and GnRH analogs do not decrease prostate size.

A 29-year-old woman has been trying for many months to become pregnant, and fertilization has just occurred following her most recent ovulation. What process will now occur that will differentiate this ovulatory cycle from those prior? A) Human chorionic gonadotropin will be produced, preventing luteal regression. B) The remaining primary follicles will provide hormonal support for the first 3 months of pregnancy. C) The corpus luteum will atrophy and be replaced by corpus albicans. D) The basal layer of the endometrium will be sloughed in preparation for implantation.

A) Human chorionic gonadotropin will be produced, preventing luteal regression. Fertilization is followed by the release of human chorionic gonadotropin. The corpus luteum, not primary follicles, provides hormonal support for early pregnancy. The atrophy of corpus luteum and its replacement by corpus albicans only occur when fertilization does not take place. The endometrium will be preserved following fertilization.

Testing for short stature growth hormone (GH) problems can be done by pharmacologic means. Which of the following medications can be utilized to test for a rise in GH? Select all that apply. A) Insulin B) Levodopa C) Persantine D) Dobutamine E) Sestamibi

A) Insulin B) Levodopa Diagnostic procedures for short stature include tests to exclude nonendocrine causes. If the cause is hormonal, extensive hormonal testing procedures are initiated. Tests can be performed using insulin, CHRH, levodopa, and arginine, all of which stimulate GH secretion so that GH reserve can be evaluated. Persantine, dobutamine, and sestamibi are used in cardiac stress testing. Sestamibi is also used in the testing of the parathyroid.

A patient with excruciating back pain that has been getting worse over the past few months comes to the Emergency Department. His chief complaint is excruciating back pain that has been getting worse over the past few months. He has also noticed decreased sensation in his lower extremities, some urinary incontinence, and feels like he never really empties his bladder all the way after voiding. Following diagnostic testing, the patient is diagnosed with advanced prostate cancer with spinal cord compression. The nurse should anticipate administering which medications to this patient? Select all that apply. A) Ketoconazole, a fungicide, to lower serum testosterone levels. B) Bisphosphonates, such as pamidronate, to inhibit bone loss C) Phosphate-binding agents such as aluminum hydroxide D) Sulfonamides with trimethoprim (Bactrim) to treat the urinary tract infection (UTI) E) Calcium carbonate to prevent osteoporosis

A) Ketoconazole, a fungicide, to lower serum testosterone levels. B) Bisphosphonates, such as pamidronate, to inhibit bone loss Inhibitors of adrenal androgen synthesis (ketoconazole) are used for treatment of patients with advanced prostate cancer who present with spinal cord compression. This is because these men need rapid decreases in their testosterone levels. The bisphosphonates (pamidronate) act by inhibiting osteoclastic activity. They prevent osteopenia, prevent and delay skeletal complications in patients with metastatic bone involvement, and provide palliation of bone pain. There is no indication that this patient has a UTI. Patients with this malignancy usually have hypercalcemia, and therefore administration of calcium supplements would be contraindicated.

Prior to performing a laparoscopy, a patient exhibiting which of the following clinical manifestations would be treated with oral antibiotic therapy for suspected pelvic inflammatory disease (PID)? Select all that apply. A) Lower abdominal pain. B) Tenderness when cervix is touched during bimanual exam. C) Feeling of fullness and bloating in abdomen. D) Purulent cervical drainage noted on tissue after voiding. E) Elevated white cell count.

A) Lower abdominal pain. B) Tenderness when cervix is touched during bimanual exam. D) Purulent cervical drainage noted on tissue after voiding. E) Elevated white cell count. The symptoms of PID include lower abdominal pain, which may start just after a menstrual period; dyspareunia; back pain; purulent cervical discharge; adnexal tenderness; and cervical motion tenderness on bimanual examination with no other apparent cause. Fever, increased erythrocyte sedimentation rate, and elevated WBC are commonly seen.

A 31-year-old male has been referred to a fertility clinic following a count of 2 million sperm/mL on seminal fluid analysis. Which of the following hormone levels would a clinician want to examine in an effort to ascertain the cause of the client's low sperm count? Select all that apply. A) Luteinizing hormone (LH) B) Adrenocorticotropic hormone (ACTH) C) Follicle-stimulating hormone (FSH) D) Cortisol E) Somatostatin F) Gonadotropin-releasing hormone (GnRH)

A) Luteinizing hormone (LH) C) Follicle-stimulating hormone (FSH) F) Gonadotropin-releasing hormone (GnRH) LH, FSH, and GnRH are all components of the hypothalamic-pituitary control of spermatogenesis. ACTH, cortisol, and somatostatin are not directly involved in the process.

While explaining milk production and release to a group of expectant parents, the nurse educates the parents on the fact that suckling by the infant provides the stimulus for milk ejection. This suckling sends feedback to the hypothalamus, which stimulates the release of which hormone from the pituitary gland responsible for the ejection of milk into the ductal system? A) Oxytocin B) Prolactin C) Progesterone D) Follicle-stimulating hormone

A) Oxytocin During lactation, milk is secreted by alveolar cells, which are under the influence of the anterior pituitary hormone prolactin. Milk ejection from the ductile system occurs in response to the release of oxytocin from the posterior pituitary. The suckling of the infant provides the stimulus for milk ejection. Suckling produces feedback to the hypothalamus, stimulating the release of oxytocin from the posterior pituitary. Oxytocin causes contraction of the myoepithelial cells lining the alveoli and ejection of milk into the ductal system.

A 40-year-old African American patient is at increased risk of developing prostate cancer, since his father was diagnosed with the disease at age 60 and his brother at age 56. What diagnostic measures should be undertaken? A) PSA and DRE starting before the age of 50 B) Transrectal ultrasonography starting immediately C) PSA and DRE starting at age 50 D) Transrectal MRI starting at age 45

A) PSA and DRE starting before the age of 50 The current recommendation of the American Cancer Society and the American Urological Association is that men at increased risk of prostate cancer, such as this patient, who has two risk factors, his race and his family history, should begin screening with an annual digital rectal exam (DRE) and prostate-specific antigen (PSA) measurement starting at age 45. The general population is advised to undergo this process starting at age 50. It is important to note that PSA levels can be a marker of either benign prostatic hyperplasia or cancer, and there is some degree of controversy regarding the benefit of screening for it. Although transrectal ultrasonography may detect small cancers, its prohibitive cost excludes its routine use in screening.

A 68-year-old male has visited his family physician because the size of his scrotum has increased exponentially in recent months. He has subsequently been diagnosed with an inguinal hernia. Which of the following statements best captures the nature of his health problem? A) Part of his intestine and parietal peritoneum are protruding through the inguinal opening. B) Rupture of the tunica vaginalis has allowed a part of his small bowel to protrude into the intrascrotal space. C) Incomplete closure of the inguinal opening during his development has contributed to testicular hypertrophy and hyperplasia. D) Occlusion of the efferent ductules and epididymis causes massive dilation of the testes.

A) Part of his intestine and parietal peritoneum are protruding through the inguinal opening. An inguinal hernia is the protrusion of intestine and peritoneum into the scrotum through an incompletely sealed inguinal opening. The tunica vaginalis, occlusion of the duct system, nor testicular hypertrophy and hyperplasia contribute to the etiology.

A 70-year-old woman who delivered four children during her reproductive years has weakened pelvic floor muscles. Which of her following anatomical structures is least susceptible to inappropriate herniation into her vagina? A) Peritoneum B) Uterus C) Bladder D) Rectum

A) Peritoneum While displacement of the uterus, bladder, and rectum can result in uterine prolapse, cystocele, and rectocele, the peritoneum is unlikely to prolapse into the vagina.

A 13-year-old female is undergoing rapid development of her breasts after experiencing menarche several months ago. Which of the following hormones are active in the development of her breasts? Select all that apply. A) Prolactin B) Human growth hormone C) Luteinizing hormone D) Estrogen E) Follicle-stimulating hormone F) Progesterone

A) Prolactin C) Luteinizing hormone D) Estrogen E) Follicle-stimulating hormone F) Progesterone In women, the pituitary release of FSH, LH, and prolactin at puberty stimulates the ovary to produce and release estrogen. This estrogen stimulates the growth and proliferation of the ductile system. With the onset of ovulatory cycles, progesterone release stimulates the growth and development of ductile and alveolar secretory epithelium.

A couple has been trying to get pregnant for over 2 years. After infertility testing, the physician has informed the male that he has a low sperm count. The nurse knows which of the following causes of low sperm count may lead to impaired spermatogenesis? Select all that apply. A) Prolonged fever B) Wearing tight briefs rather than boxers C) Undescended testes D) Swimming in cold-water lakes E) Vacationing in a tropical location

A) Prolonged fever B) Wearing tight briefs rather than boxers C) Undescended testes Prolonged exposure to elevated temperatures, as a result of prolonged fever, tight undergarments, and failure of the testes to descend into the scrotum can result in decreased sperm counts.

A nurse who works on a urology-gynecology ward of a hospital is coming on shift and will be caring for a 34-year-old woman who has been admitted overnight for the treatment of a large endometriosis. What interventions should the nurse most realistically anticipate providing over the course of the shift and the next several days? A) Providing pain control; preparing the client for a laparoscopic procedure or hysterectomy. B) Administration of packed red blood cells to compensate for low hemoglobin; administering hormone therapy. C) Assisting with a Pap smear; administration of high-dose corticosteroids. D) Administration of male androgens to minimize endometrial hyperplasia; facilitating a dilation and curettage.

A) Providing pain control; preparing the client for a laparoscopic procedure or hysterectomy. Pain control is central to treatment of endometriosis, and surgical treatment is ideally performed laparoscopically, though hysterectomy is sometimes indicated. Bleeding is not a common symptom of endometriosis, and neither corticosteroids nor male androgens are common treatments.

Which of the following statements best captures the essence of a second messenger in the mechanisms of the endocrine system? A) Second messengers act as the intracellular signal that responds to the presence of a hormone. B) Endocrine-producing cells must release both a hormone and a second messenger in order to exert a distant effect. C) Second messengers act to supplement hormone effects on cell receptors when the desired hormonal effect must be either increased. D) Second messengers provide an alternative pathway for endocrine effects on a cell that bypass the normal receptor pathways

A) Second messengers act as the intracellular signal that responds to the presence of a hormone. Second messengers interact with hormones that cannot cross the cell membrane, and they mediate the ultimate effect on the cell. They are not produced by the hormone-producing cell, and they are necessary to bring about hormonal effects, not simply for increasing the intensity of the effect. They are not an alternative mechanism of effect but rather a prerequisite for certain hormonal effects on body cells.

A 34-year-old male has been diagnosed with testicular cancer after he visited his family physician with a complaint of an enlarged, painful testicle. Biopsy has indicated that his tumor is malignant, and his oncologist believes that the tumor arose in the seminiferous epithelium and is producing a uniform population of cells. What is the client's most likely specific diagnosis? A) Seminoma B) Choriocarcinoma C) Germ cell tumor D) Nonseminoma

A) Seminoma Seminomas are the subtype of germ cell tumors that are most common in the fourth decade and are thought to originate with the seminiferous epithelium and produce a uniform cell population. Nonseminomas tend to produce a variety of cell types, and choriocarcinoma is rare variant of nonseminoma testicular cancer that originates in the placental tissue.

Which of the following statements best captures an aspect of the role of hormones in the body? A) Some chemical substances can function as hormones or be integrated with the central and peripheral nervous systems. B) Hormones directly initiate many of the processes that contribute to homeostasis. C) Control of body processes is ensured by the fact that a single hormone can only exert one effect on one specific system or tissue. D) Each hormone that exists in the body is produced by only one specific endocrine gland.

A) Some chemical substances can function as hormones or be integrated with the central and peripheral nervous systems. Some chemicals, such as epinephrine, can both function as a hormone and be closely integrated with the central and peripheral nervous systems as well as the immune systems, leading to current terminology such as "neuroendocrine." Hormones modulate, but do not initiate, changes in the body, and one hormone may exert multiple effects on multiple body systems. Hormones are produced by a variety of body tissues, not solely by endocrine glands.

A 38-year-old male has presented to a clinic for the treatment of severe dermatitis after contact with poison ivy on a camping trip. The client has been prescribed prednisone, a corticosteroid, for the treatment of his skin condition. The client's care provider has emphasized that dosages of the drug will be gradually tapered off rather than stopped upon resolution of the symptoms. What is the most accurate rationale for this dosing protocol? A) The client's hypothalamic-pituitary-adrenal (HPA) system will require recovery time before normal function is restored. B) Steroids can induce a dependency that it best addressed with a gradual withdrawal. C) HPA function is heightened during steroid administration and must return to normal levels before the drug is completely stopped. D) Abrupt cessation of the drug can contribute to symptoms similar to Cushing syndrome.

A) The client's hypothalamic-pituitary-adrenal (HPA) system will require recovery time before normal function is restored. The suppression of the HPA system that accompanies steroid therapy requires time for a return to normal function. Dependency on the drug itself is not the rationale for tapering, and HPA function is suppressed, not heightened during therapy. Abrupt cessation can contribute to an Addison disease-like response, not Cushing syndrome.

A 48-year-old woman has been found to have nodules on her thyroid that must be biopsied to determine whether or not they are malignant. Which of the following imaging techniques will be most helpful to the surgeon in visualization of the nodes for fine needle aspiration? A) Ultrasound B) Magnetic resonance imaging C) Radioactive scanning using radioiodine D) Radioactive scanning using sestamibi

A) Ultrasound Thyroid ultrasound is recommended for managing thyroid nodules and can aid in visualization of the nodule for biopsy (fine needle aspiration [FNA]), which is necessary to help distinguish benign from malignant etiology. Magnetic resonance imaging is the preferred choice for pituitary and hypothalamic imaging. Isotopic imaging includes radioactive scanning of the thyroid (e.g., using radioiodine), parathyroids (e.g., using sestamibi), and adrenals (e.g., using metaiodobenzylguanidine [MIBG] to detect pheochromocytoma).

While teaching a class on female cancers, the instructor emphasizes to the nursing students that many patients with ovarian cancer may display A) abdominal pain, bloating, feeling full quickly after ingesting food. B) pain after intercourse, bleeding irregularities, perineal tenderness. C) colicky low abdominal pain, adnexal mass present without palpation. D) lower abdominal pain localized to one side, referred shoulder pain.

A) abdominal pain, bloating, feeling full quickly after ingesting food. Symptoms that are believed to have a strong correlation to ovarian cancer include abdominal pain, increased abdominal size or bloating, and difficulty in eating or feeling full quickly after ingesting food. Pain and bleeding after intercourse are usually associated with an inflammatory process. Colicky low abdominal pain and adnexal mass are associated with cancer of the fallopian tube. Localized abdominal pain to one side with referred shoulder pain is common in ectopic pregnancy.

A 66-year-old man has presented to a nurse practitioner to get a refill for his antiplatelet medication. The client has a history of ischemic heart disease and suffered a myocardial infarction 5 years ago and has unstable angina; he uses a transdermal nitroglycerin patch to control his angina. The client has a 40 pack-year smoking history and uses nebulized bronchodilators at home for the treatment of transient shortness of breath. He has long-standing hypertension that is treated with a potassium-sparing diuretic and a-adrenergic-blocking medication. During the nurse's assessment, the man states that he has been unable to maintain his erection in recent months. Which of the following aspects of the man's health problems and treatments would the nurse identify as contributing to his erectile difficulty (ED)? Select all that apply. His

A) antihypertensive medications. C) hypertension. E) smoking history. F) age. Hypertension, antihypertensive medications, age, and smoking are all implicated in the etiology of ED. Ischemic heart disease and bronchodilators are less likely to directly contribute to the condition.

Neurotransmitters like catecholamines (e.g., dopamine and epinephrine) have a reaction time of A) milliseconds. B) less than 10 minutes. C) 24 to 36 hours. D) 4 to 7 days.

A) milliseconds. The neurotransmitters, which control the opening of ion channels, have a reaction time of milliseconds.

Which of the following best describes the half-life of a highly protein-bound drug such as thyroxine (99% protein bound)? The half-life would be A) much longer to reduce the concentration of the hormone by one half. B) shorter because only a little of the hormone has to be used up to reduce the concentration. C) dependent on which drugs were in the blood system holding on to the hormone. D) dependent on the liver to carry the hormone to its designated target organ.

A) much longer to reduce the concentration of the hormone by one half. The half-life of a hormone—the time it takes for the body to reduce the concentration of the hormone by one half—is positively correlated with its percentage of protein binding. Thyroxine, which is more than 99% protein bound, has a half-life of 6 days, whereas aldosterone, 15% bound, has a half-life of only 25 minutes.

Following a workup that included endocrine studies (FSH/LH, prolactin, testosterone, DHEAS levels), a 22-year-old college student complaining of abnormal bleeding has been diagnosed with dysmenorrhea due to alterations in her hormone levels. The nurse should anticipate that she will likely be prescribed: Select all that apply. A) oral contraceptives. B) estrogen only. C) prostaglandin synthetase inhibitors. D) anxiolytic medications. E) androgens.

A) oral contraceptives. C) prostaglandin synthetase inhibitors. The treatment of dysfunctional bleeding of a nonhormonal nature is usually treated with oral contraceptives or cyclic progesterone therapy. Prostaglandin synthetase inhibitors are prescribed for dysmenorrhea. Anxiolytic drugs treat mood changes of PMS.

A 28-year-old male who is 6'11'' tall has a diagnosis of acromegaly. The man is explaining to a curious but sympathetic coworker exactly what accounts for his extraordinary height. Which of the following explanations demonstrates a sound understanding of his health problem? A) "My pituitary gland produced a much higher than normal amount of growth hormone when I was a child." B) "A tumor in my brain threw off my hormone levels after I was finished adolescence." C) "My liver is malfunctioning and produces too many of the hormones that ultimately cause growth." D) "The high sugar levels that go along with my diabetes made my pituitary gland overproduce the hormones that cause you to grow."

B) "A tumor in my brain threw off my hormone levels after I was finished adolescence." Acromegaly is associated with adult onset and nearly always involves an adenoma. Increased GH as a child and liver dysfunction are not noted contributors to acromegaly. High levels of GH can cause overproduction of insulin and eventual diabetes, but diabetes does not itself lead to acromegaly.

A medical student is assessing a 22-year-old male who has come to the emergency department because of pain and swelling in his scrotum over the past 36 hours. The attending physician has told the student that she suspects epididymitis. Which of the medical student's following questions is most likely to be useful in the differential diagnosis of epididymitis? A) "Do you know if your vaccinations for mumps are up to date?" B) "Have you had unprotected sex in the past?" C) "Do you have a sensation of heaviness in the left side of your scrotum?" D) "Has it been painful when you get an erection lately?"

B) "Have you had unprotected sex in the past?" Sexually transmitted infections are a common contributing factor to epididymitis. Mumps are more often a precursor to orchitis, and heaviness on the left side is associated with varicocele. Pain with an erection is not a noted symptom of epididymitis.

After hearing daunting reports from her slightly older coworkers and friends, a 44-year-old woman has a number of questions for her physician about what to expect during perimenopause and why. Which of the following teaching points is most accurate? A) "The decrease in estrogen and most other hormones in your body do cause a lot of instability for most women around menopause." B) "Hot flashes are a reality for most women in menopause, and the exact cause of them isn't known yet." C) "The emotional swings that often accompany menopause result from changes to the limbic center in the brain, which governs emotion." D) "There are a lot of changes to the gastrointestinal and respiratory systems that exist around menopause that ultimately result from estrogen deficiency."

B) "Hot flashes are a reality for most women in menopause, and the exact cause of them isn't known yet." Hot flashes are a common accompaniment to menopause, and their exact etiology is not known. Menopause is caused by a gradual reduction in ovarian estrogen production, but decreases in other hormones do not commonly occur. Organic brain changes are not a noted component of menopause, and GI and respiratory symptoms are not prevalent.

A 20-year-old male has been diagnosed with testicular cancer and is seeking information about his diagnosis from a number of Web sites. Which of the following statements that he has read is most plausible? A) "Testicular cancer is a leading cause of death among males who should be in the prime of their life." B) "Men with cryptorchidism--the term for an undescended testicle--are known to have a higher risk of developing testicular cancer." C) "For most men with testicular cancer, bloody urine is their first sign that something is wrong." D) "Recent developments in the treatment of testicular cancer mean that few men now need to have a testicle removed."

B) "Men with cryptorchidism--the term for an undescended testicle--are known to have a higher risk of developing testicular cancer." Cryptorchidism is an identified risk factor for the development of testicular cancer. The diagnosis is no longer a leading cause of death. Hematuria is not a common symptom, and orchiectomy is still the standard treatment modality.

During a group prenatal class, the nurse teaching the course is explaining the thinning of the cervix that accompanies the stages of labor leading up to delivery. A participant admits that she is not familiar with either the location or normal role of the cervix. Which of the following responses by the nurse would be most appropriate? A) "Your cervix is just the term for the lowest part of your uterus, and it is tightly shut at all times except in woman who are giving birth." B) "The cervix is the opening between the uterus and the vagina, and so it opens wide during the lead-up to birth." C) "Your cervix is the part of your uterus where the fallopian tubes join in and the baby has to pass through it to the vagina." D) "The cervix is the soft lining of your uterus that has to expand greatly to accommodate your baby during delivery."

B) "The cervix is the opening between the uterus and the vagina, and so it opens wide during the lead-up to birth." The cervix is best characterized as the neck of the uterus that projects into the vagina. It opens selectively to accommodate menses. The fallopian tubes enter the uterus far above the cervix, and the lining of the uterus is known as the endometrium.

A 60-year-old woman who is 11 years menopausal has presented to the emergency department stating, "I haven't had my period in years, but lately I've been bleeding again, and quite heavily in the last few days." The care team needs to rule out endometrial cancer. How should they best explain the most accurate plan for confirming or ruling out the diagnosis? A) "We're going to book a CT scan for you as soon as possible so that we can see what is inside your uterus." B) "We're going to have to open your cervix with a speculum and take scrapings from the wall of your uterus." C) "We are going to order blood work that will measure your hormone levels." D) "We can do a Pap smear right now, and we will get the results as soon as we can."

B) "We're going to have to open your cervix with a speculum and take scrapings from the wall of your uterus." D&C is a more accurate diagnostic procedure for endometrial cancer than CT, Pap smear, or blood work analysis.

While educating a mother about the benefits of giving her child the human papilloma virus (HPV) vaccine, gardisil, which of the following statements will the nurse need to clarify for the parent? Select all that apply. The vaccine is A) best administered before the child becomes sexually active. B) 100% effective against development of cervical cancer for her life span. C) effective against the two most common strains of genital warts. D) only recommended for females between the ages of 9 and 26. E) recommended prior to exposure to HPV, and if the child has genital warts, they are already exposed.

B) 100% effective against development of cervical cancer for her life span. D) only recommended for females between the ages of 9 and 26. The HPV vaccine has decreased the risk of cervical cancer by 97%. Gardisil is one type of HPV vaccine to prevent infection with the HPV subtypes 16, 18, 9, and 11. This vaccine has been approved for girls and boys between 9 and 26 years of age (prior to becoming sexually active) to prevent HPV 6 and 11 genital warts. The vaccine targets the two strains of HPV (16 and 18) responsible for 70% of the cervical cancer. There is no treatment that is 100% effective against cervical cancer.

Which of the following sexually active women most likely faces the highest risk of developing an ectopic pregnancy? A) A 14-year-old who experienced menarche 2 years prior. B) A 42-year-old who has decided to try to have one more child and has had her tubal ligation reversed. C) A 27-year-old who stopped using medroxyprogesterone contraceptive injection (Depo-Provera) several months ago. D) A 22-year-old who has a history of anorexia nervosa and who has a body mass index (BMI) of 12.0 (normal weight = 18.5 to 24.9).

B) A 42-year-old who has decided to try to have one more child and has had her tubal ligation reversed. Previous tubal ligation is an identified risk factor for ectopic pregnancy. Young age, use of injectable contraception, and low BMI are not specifically associated with ectopic pregnancy.

A patient exhibiting problems with his or her thyroid has been scheduled for a radioactive scan. From the following list of patients, what would the nurse question as to whether this would be a safe procedure for this patient? A) An adult patient having an episode of wheezing from allergies B) A young female patient who has been trying to get pregnant C) A middle-aged male patient with uncontrolled type 2 diabetes mellitus D) An elderly patient who has a history of aortic stenosis

B) A young female patient who has been trying to get pregnant Radioactive iodine therapy is contraindicated in pregnant women because 131I crosses the placenta and can adversely affect the fetal thyroid gland. The other clients would have no contraindication to the substance.

Following a meal, a woman's blood glucose level has increased. In addition, her pancreas has increased the amount of insulin produced and released. Which of the following phenomena has occurred? A) Increased hormone level according to a negative feedback mechanism B) Adjustment according to the level of the substance a hormone regulates C) Hormone production and release via the positive feedback cycle D) Hypothalamic-pituitary control of hormone levels

B) Adjustment according to the level of the substance a hormone regulates The level of some hormones is adjusted according to the amount of the substance that they control. In this case, insulin controls glucose levels and would increase in response to the increase in serum glucose that follows a meal. This differs from a negative feedback cycle in which a simple decrease in a hormone level stimulates production and/or release of that hormone. Positive feedback and hypothalamic-pituitary control are not evident in this situation.

Following a long history of intermittent back pain and urinary urgency, a 50-year-old client has been diagnosed with chronic bacterial prostatitis. Which of the following factors is most likely to influence his health care provider's choice of treatment? A) The diagnosis is thought to have an autoimmune etiology with limited response to steroid treatments. B) Antibacterial drugs penetrate poorly into the chronically inflamed prostate. C) Urethral catheterization provides symptom relief and contributes to resolution of the underlying infection. D) There are no proven treatments for chronic prostatitis that address the infectious process.

B) Antibacterial drugs penetrate poorly into the chronically inflamed prostate. The fact that antibacterial drugs penetrate poorly into the chronically inflamed prostate makes treatment difficult. Inflammatory prostatitis, not chronic prostatitis, is believed to be an autoimmune problem. Urethral catheterization does not provide resolution of the underlying infection, though treatment modalities do exist.

After a long and frustrating course of constant vaginal pain, a 38 y/o F has diagnosed with generalized vulvodynia by her gynecologist. What treatment plan is her physician most likely to propose? A) Alternative herbal therapies coupled with antifungal medications B) Antidepressant and antiepileptic medications C) Lifestyle modifications aimed at accommodating and managing neuropathic pain D) Narcotic analgesia and nonsteroidal anti-inflammatory medications

B) Antidepressant and anti epileptic medications Treatment of vulvodynia necessitates a long-term, chronic pain approach; antidepressants & anti epileptic medications are often used. Alt: standard analgesic regimens and lifestyle changes

A newborn male has been diagnosed with hypospadias follwoing his postpartum assessment by a pediatrcian. Which of the follwoing diagnostics and treatment options is the physician most likely to rule out first? A) Chromosomal studies B) Circimcision C) Surgical repair D) Testosterone supplementation

B) Circumcision Because the foreskin is often used in surgical repair of hypospadias, circumcision is normally contraindicated. Chromosomal studies are frequently warranted, and surgery is the standard treatment for the repair of the urethra. Testosterone supplementation is often necessary.

When explaining to a patient why his prostate is enlarging, the nurse will mention that which of the following hormones may contribute to the prostatic hyperplasia? Select all that apply. A) Glucocorticoids B) Testosterone C) Dihydrotestosterone D) Estrogens E) Progesterone

B) Testosterone C) Dihydrotestosterone D) Estrogens Both androgens (testosterone and dihydrotestosterone) and estrogens appear to contribute to the development of BPH. Testosterone is the most important factor for prostatic growth. DHT, the biologically active metabolite of testosterone, is thought to be the ultimate mediator of prostatic hyperplasia, with estrogen serving to sensitize the prostatic tissue to the growth-producing effects of DHT. Glucocorticoids do not play a role in making the prostate gland enlarge.

A nursing student studying obstetrics asks the faculty, "How does the sperm get so much energy to swim up the fallopian tubes?" The instructor responds based on which pathophysiological principle? A) Usually the ejaculation is so powerful that it propels the sperm far up into the fallopian tubes. B) Fructose is secreted by the seminal vesicles and provides energy for motility of the sperm. C) The seminal vesicles secrete fluid for the semen, and they can secrete catecholamines that energize the sperm. D) Estrogen, which is being secreted when the ova are ripe and ready for penetration, assists by attracting the sperm in the right direction.

B) Fructose is secreted by the seminal vesicles and provides energy for motility of the sperm. The seminal vesicles consist of two highly tortuous tubes that secrete fluid for the semen. Each of the paired seminal vesicles is lined with secretory epithelium containing an abundance of fructose, prostaglandins, and several other proteins. The fructose secreted by the seminal vesicles provides the energy for sperm motility.

Following the identification of low blood levels of cortisol and low 24-hour urinary free cortisol, a 51-year-old female client has been diagnosed with a primary adrenal cortical insufficiency. Which of the following health consequences would be attributable to her low levels of cortisol? A) Visible exophthalmos B) Impaired immunological and inflammatory response C) Diminished secondary sex characteristics D) Insufficient regulation of serum potassium and sodium lev

B) Impaired immunological and inflammatory response Cortisol plays a central role in the normal functioning of the immune response and inflammation. Exophthalmos is associated with Graves disease, and secondary sex characteristics are functions of adrenal androgens. Potassium and sodium are regulated by mineralocorticoids.

An infant born with congenital hypothyroidism and has not sought care from any health care provider is likely to develop which of the following complications? Select all that apply. A) Deformed joints and bone spurs B) Impaired physical growth C) Mental retardation D) Loss of fine motor control and arthritis E) Down syndrome

B) Impaired physical growth C) Mental retardation Thyroid hormone is essential for normal growth and brain development, almost half of which occurs during the first 6 months of life. If untreated, congenital hypothyroidism causes mental retardation and impairs physical growth. Down syndrome is a congenital birth defect and not caused by hypothyroidism.

The nurse knows that when combined continuous estrogen-progesterone therapy (CCEPT) is prescribed, the drug is considered effective if which of the following occurs? A) More regular periods for women with irregular menses B) Inhibited endometrial development resulting in no menses C) Relaxation of the myometrium, thereby limited painful cramps D) Shedding of the endometrial build-up on a more regular basis

B) Inhibited endometrial development resulting in no menses Thus, continuous exposure to progesterone inhibits endometrial development. Eventually, the combined continuous estrogen-progesterone therapy (CCEPT) results in no bleeding. It can be associated with irregular bleeding and spotting until the lining becomes atrophic. It relieves cramps rather than generating them.

A 20-year-old female has come to the clinic complaining of severe menstrual cramps. The clinic nurse practitioner knows that dysmenorrhea occurs when which muscle group contracts? A) Perimetrium B) Myometrium C) Endometrium D) Fundus

B) Myometrium The middle muscle layer, the myometrium, forms the major portion of the uterine wall. Contractions of these muscle fibers help to expel menstrual flow and the products of conception during miscarriage or childbirth. When pain accompanies the contractions associated with menses, it is called dysmenorrheal. The perimetrium is the outer layer of the uterus. Endometrium, the inner layer of the uterus, is made up of a basal and a superficial layer. The superficial layer is shed during menstruation and regenerated by cells of the basal layer. The fundus is the portion of the uterus that lies about the insertion of the fallopian tubes.

A 31-year-old patient with breast cancer is concerned about being prescribed a hormonal medication to help block the effects of estrogen on the growth of breast cancer cells. The nurse should provide education about which of the following likely medications? A) Trastuzumab (Herceptin), a biologic therapy B) Nolvadex (Tamoxifen), a nonsteroidal antiestrogen C) Progesterone, a hormone. D) Arimidex (anastrozole), an aromatase inhibitor.

B) Nolvadex (Tamoxifen), a nonsteroidal antiestrogen Hormone therapy is used to block the effects of estrogen on the growth of breast cancer cells. Tamoxifen is a nonsteroidal antiestrogen that binds to estrogen receptors and blocks the effects of estrogens on the growth of malignant cells in the breast. Herceptin is used to stop the growth of breast tumors that express the HER2/neu receptor on their cell surface. Arimidex, an aromatase inhibitor, blocks the enzyme that converts androstenedione and testosterone to estrogen in the peripheral tissues. This reduces the circulating estrogen levels in postmenopausal women.

Following destruction of the pituitary gland, ACTH stimulation stops. Without ACTH to stimulate the adrenal glands, the adrenals' production of cortisol drops. This is an example of which type of endocrine disorder? A) Primary B) Secondary C) Tertiary D) Somatic

B) Secondary In secondary disorders of endocrine function, the target gland is essentially normal, but defective levels of stimulating hormones or releasing factors from the pituitary system alter its function.

The nurse knows that if which of the following hormones listed below are not available during embryonic development, a male embryo with an XY chromosomal pattern may develop female external genitalia? Select all that apply. A) Anti-Müllerian hormone (AMH) B) Testosterone C) Dihydrotestosterone (DHT) D) Acrosome E) Follicle-stimulating hormone (FSH)

B) Testosterone C) Dihydrotestosterone (DHT) In the absence of testosterone (and DHT), a male embryo with an XY chromosomal pattern develops female external genitalia. AMH suppresses the Müllerian ducts and prevents development of the uterus and fallopian tubes in the male. Acrosome is the outside of the anterior two thirds of the head of a sperm. For spermatogenesis to occur, FSH binds to specific receptors in Sertoli cells.

Which of the following statements best captures the role of the adrenal cortex in maintaining homeostasis? A) The adrenal cortex is responsible for the production of epinephrine and norepinephrine that are part of the sympathetic nervous system. B) The adrenal cortical hormones are primarily steroids and sex hormones. C) Redundant, secondary production of adrenal cortical hormones can compensate for the loss of the adrenal glands. D) Normal sexual function is dependent on adequate adrenal cortical function.

B) The adrenal cortical hormones are primarily steroids and sex hormones. The adrenal cortex is responsible for secreting three types of hormones: the glucocorticoids, the mineralocorticoids, and the adrenal androgens. The adrenal medulla produces epinephrine and norepinephrine, and there are no alternate production sites for adrenal cortical hormones. The adrenal androgens are least responsible for normal sexual function.

During the follicular stage of menstruation, increased estradiol production causes an increase in FSH production. This increase in FSH production by the anterior pituitary gland will have what effect on the follicle? A) The follicle will continue to grow until it can no longer stay in its membrane. B) The follicle will die, which results in a fall of FSH. C) The follicle will continue to grow and produce estradiol. D) The follicle will secrete additional hormones to attract swimming sperm.

B) The follicle will die, which results in a fall of FSH. In positive feedback control, rising levels of a hormone cause another gland to release a hormone that is stimulating to the first. Increased estradiol production during the follicular stage of the menstrual cycle causes increased FSH production by the anterior pituitary gland. This stimulates further increases in estradiol levels until the demise of the follicle.

A middle-aged woman has acromegaly as a result of a pituitary adenoma that was found and removed when she was a teenager. The physician is suspecting that the tumor has returned and has ordered a diagnostic work-up. A glucose load is ordered. If the tumor has returned, the nurse would expect which of the following results? A) The glucoses load will suppress GH level. B) The growth hormone level will not be suppressed following glucose load. C) The glucose load will raise her serum glucose level to the point of requiring insulin. D) There will be no change in the serum growth hormone level following the glucose load.

B) The growth hormone level will not be suppressed following glucose load. When a GH-secreting tumor is suspected, the GH response to a glucose load is measured as part of the diagnostic workup. Normally, a glucose load would suppress GH levels. However, in adults with GH-secreting tumors (acromegaly), GH levels are not suppressed (and paradoxically increase in 50% of cases) to a glucose load.

Following a long history of fatigue, weakness, and poor appetite, a 39-year-old male has been diagnosed with hypopituitarism. Which of the following clinical findings would most likely cause his care team to suspect that the man has an additional endocrine disorder from a different source? A) The man has a low sperm count and has been unable to have children. B) The man has a chronic platelet deficiency and is occasionally anemic. C) The client is 52 tall and was consistently short for his age as a child. D) The man displays the signs and symptoms of hypothyroidism.

B) The man has a chronic platelet deficiency and is occasionally anemic. Low platelets and low hemoglobin are unlikely to be a manifestation of hypopituitarism. A low sperm count, small stature, and hypothyroidism are all noted manifestations of pituitary hypofunction.

Which of the following statements best captures an aspect of the function of the hypothalamic-pituitary-adrenal (HPA) system? A) Adrenocorticotropic hormone (ACTH) released by the hypothalamus controls to release of cortisol. B) The pituitary gland communicates with the adrenal cortex through the release of ACTH. C) The adrenal cortex receives corticotrophin-releasing hormone (CRH) and in turn releases cortisol. D) The pituitary gland causes a release of CRH from the hypothalamus, which promotes hormone release from the adrenal cortex.

B) The pituitary gland communicates with the adrenal cortex through the release of ACTH. ACTH mediates between the anterior pituitary gland and the adrenal cortex in the HPA system. ACTH is released by the pituitary, not the hypothalamus, and CRH acts on the pituitary, not the adrenal cortex. CRH flows from the hypothalamus to the pituitary, not vice versa.

A woman consults her physician about a study (Women's Health Initiative) discussing an increased risk for breast cancer if taking hormones. The physician can allay her fears by sharing which of the following findings? A) Only women with a family history of breast cancer had an increased risk for developing breast cancer while on birth control pills. B) This 26% increased risk for developing invasive breast cancer occurred in women taking combined continuous estrogen-progesterone therapy. C) The increased risk for developing inflammatory breast cancer occurred in women taking hormone replacement therapy when they were over the age of 45. D) Follow-up studies demonstrated that there was only a 3% increase in risk for developing breast cancer if hormones were discontinued immediately.

B) This 26% increased risk for developing invasive breast cancer occurred in women taking combined continuous estrogen-progesterone therapy. The WHI added to the breast cancer concern by reporting a 26% increased risk of invasive breast cancer in the women using CCEPT. Results from a 3-year WHI follow-up study revealed that the breast cancer risk of women who stopped taking CCEPT continued at a rate similar to that observed during the intervention.

A 13-year-old patient undergoing puberty is alarmed to find small, white bumps surrounding the areolae on her nipples. You reassure her that these are a normal sign of her sexual maturation and tell her that they are which of the following? A) Skene glands B) Bartholin glands C) Montgomery tubercles D) Cooper ligaments

C) Montgomery tubercles The small bumps or projections on the areolar surface known as Montgomery tuberclesare sebaceous glands that keep the nipple area soft and elastic. At puberty and during pregnancy, increased levels of estrogen and progesterone cause the areola and nipple to become darker and more prominent and the Montgomery glands to become more active.

A client with a new diagnosis of an endocrine disorder is unclear how the body can control the levels of different hormones over time. Which of the following statements most accurately underlies the dominant regulation process of hormone levels in the body? A) A positive feedback cycle ensures that stable levels of hormones exist in the body over time. B) With input from various sensors, hormone production and release are adjusted based on existing hormone levels. C) The hypothalamus ensures that hormone levels correspond accurately to the diurnal cycle. D) The pituitary gland is genetically programmed to stimulate and inhibit hormone production and/or release based on the needs at different points in the life cycle.

B) With input from various sensors, hormone production and release are adjusted based on existing hormone levels. Most hormone levels are controlled by way of a negative feedback cycle, in which low levels stimulate production and/or release. A positive feedback cycle would not achieve this effect. While some hormones are released on a diurnal schedule, the dominant form of hormone regulation in the body is that of negative feedback. Hormone release is not predetermined by the pituitary gland.

A nurse practitioner has a 30-year-old male patient presenting with fever and chills, urinary frequency and urgency, and pain with urination. A urine sample displays cloudy and foul-smelling urine. During digital rectal exam of the prostate, the nurse notes a thick white discharge. A likely diagnosis would be A) urinary tract infection requiring a prescription of sulfur drugs. B) acute bacterial prostatitis requiring antimicrobial therapy. C) testicular cancer requiring appointment with an urologist. D) inflammation of the epididymis requiring scrotal elevation.

B) acute bacterial prostatitis requiring antimicrobial therapy. Manifestations of acute bacterial prostatitis include fever and chills, malaise, frequent and urgent urination, and dysuria. The urine may be cloudy and malodorous because of urinary tract infection. Rectal examination reveals a swollen, tender prostate. During exam, prostatic massage produces a thick discharge with WBCs that grow a large numbers of pathogens on culture.

An elderly male patient is complaining of dribbling after he urinates and feeling like he never empties his bladder. The nurse suspects the patient may have a problem with A) kidney stones. B) an enlarged prostate gland. C) blood clots clogging the urethra. D) calcium sediment in the bladder.

B) an enlarged prostate gland. Symptoms of obstruction, including dribbling after urinating and the continuous sensation of having to urinate, are most likely due to enlargement of a prostate gland. Kidney stones cause intense flank pain; blood clots are usually associated with trauma or surgery; calcium sediment could imply a kidney stone or be a result of a procedure to break up kidney stones into smaller segments.

While teaching a health class to junior and senior male high school students, the school nurse educates them regarding the first sign of testicular cancer, which would include A) one testicle being lower than the other in the scrotal sac. B) enlargement of the testicle. C) back pain. D) coughing bloody sputum.

B) enlargement of the testicle. Often the first sign of testicular cancer is a slight enlargement of the testicle that may be accompanied by some degree of discomfort. Back pain and hemoptysis are last signs leading to a metastatic lesion. It is normal for the testicle to hang lower than the other.

If a male has a history of impaired blood flow in his vascular system, he may be prone to develop A) premature ejaculation. B) erectile dysfunction. C) penile engorgement. D) vascular clots in pudendal arteries.

B) erectile dysfunction. Vascular disease affects male potency because it may impair blood flow to the pudendal arteries or their tributaries, resulting in loss of blood volume with subsequent poor distention of the vascular spaces of erectile tissue. Premature ejaculation is not caused by a vascular problem. Penile engorgement is expected as part of an erection. Vascular clots may relate to a trauma or surgery to the penis.

The cremaster muscles work in concert with the pampiniform plexus that surrounds the testicular artery to A) move the testes into the scrotum. B) maintain testicular temperature. C) empty fluid from the seminal vesicles into the genital ducts. D) move sperm from the ampulla to the penis.

B) maintain testicular temperature. The location of the testes in the scrotum is important for sperm production, which is optimal at 2°C to 3°C below body temperature. Two systems maintain the temperature of the testes at a level consistent with sperm production. One is the pampiniform plexus of testicular veins that surround the testicular artery. This plexus absorbs heat from the arterial blood, cooling it as it enters the testes. The other is the cremaster muscles. These muscles respond to decreases in testicular temperature by moving the testes closer to the body.

A 51-year-old woman who has been receiving estrogen and progesterone therapy (EPT) for the last 5 years has visited her care provider because her peers have told her about the risks of heart disease, stroke, and breast cancer that could accompany hormone therapy (HT). How should her care provider respond to her concerns? A) "There is a demonstrable increase in breast cancer risk with HT, but the risk of stroke or heart disease actually go down slightly." B) "All considered, the benefits of HT outweigh the slightly increased risks of heart disease, stroke, or breast cancer." C) "HT is actually associated with a decrease in heart disease risk, but there is an increase in stroke risk; the breast cancer connection is still unclear." D) "There's in fact a slight protective effect against stroke associated with HT, but this is partially offset by increased rates of heart disease and breast cancer."

C) "HT is actually associated with a decrease in heart disease risk, but there is an increase in stroke risk; the breast cancer connection is still unclear." Current evidence shows a decrease in coronary heart disease (CHD) risk with HT but an increase in CVA risk; the evidence regarding breast cancer is still indefinite.

Following a visit to her campus medical clinic motivated by persistent abdominal pain and dyspareunia, a 20-year-old female college student has been referred for a diagnostic workup to rule out pelvic inflammatory disease. Her elevated white cell and C-reactive protein levels lead her care provider to suspect pelvic inflammatory disease (PID). What follow-up question is most likely to help with the differential diagnosis? A) "Are you using tampons during your period or do you normally use pads?" B) "What does your daily hygiene routine usually consist of?" C) "How many sexual partners have you had?" D) "Have you ever had a therapeutic abortion in the past?"

C) "How many sexual partners have you had?" Having multiple sex partners is a factor that has been identified in the development of PID. The use of tampons, inadequate hygiene, and a history of TA are less likely to predispose to PID.

A patient who is still breast-feeding her infant has arrived at the clinic complaining of sore breasts. The physician has diagnosed mastitis. Which of the following statements made by the patient will require correction by the nurse? A) "They tell me this is usually caused by a Staph infection." B) "I need to make sure to wash my hands thoroughly before touching my breasts." C) "I need to stop breast-feeding until all my antibiotics have been taken." D) "A clogged duct in my breast has become clogged."

C) "I need to stop breast-feeding until all my antibiotics have been taken." Mastitis is inflammation of the breast. It most frequently occurs during lactation but also can result from other conditions. In the lactating woman, inflammation results from an ascending infection that travels from the nipple to the ductile structures. The most common organism isolated is Staphylococcus. The offending organism originates from the suckling infant's nasopharynx or the mother's hands. Infection and inflammation cause obstruction of the ductile system. The breast becomes hard and, inflamed. It is advisable for the mother to continue breast-feeding during antibiotic therapy to prevent this.

A 41-year-old male has presented to his family physician stating that for the last 2 years his erection "is as crooked as a dog's hind leg and hurts too." He has subsequently been diagnosed with Peyronie disease. Which of the following statements by his physician is most appropriate? A) "Even though it's obviously distressing, you should know that this will likely resolve on its own with time." B) "I'll refer you to a urologist who will likely want to perform a circumcision." C) "This might need surgery, but it could possibly disappear over time without needing treatment." D) "There are things you can do to minimize this, such as getting more exercise, stopping smoking, and maintaining a healthy diet."

C) "This might need surgery, but it could possibly disappear over time without needing treatment." While spontaneous resolution is possible, surgery is sometimes necessary for the treatment of Peyronie disease. Circumcision would not be an effective treatment, and lifestyle factors are not noted to contribute to the etiology or resolution.

A 68-year-old man has revealed to his care provider that he is distraught over his increasingly frequent inability to maintain an erection. Which of the following teaching points is most warranted? A) "Above the age of 65, erectile dysfunction is largely inevitable and untreatable." B) "With lifestyle modifications like exercise, weight loss, and quitting smoking, you can most likely correct this and prevent it in the future." C) "While this problem is often a part of the aging process, it might be a result of some of the medications that you take." D) "Most often this problem is transient and will resolve with time."

C) "While this problem is often a part of the aging process, it might be a result of some of the medications that you take." The incidence and prevalence of ED increase with age, and medications often contribute to the problem. ED is not inevitable or untreatable, but it is unlikely to spontaneously resolve. Lifestyle modifications can lessen the risk of developing ED but cannot guarantee correction or prevention.

A 24-year-old male patient with a complete transection of the spinal cord asks, "Will I be able to father children in the future?" The nurse responds, A) "Probably not since your spinal cord has been completely severed." B) "An urologist may be able to insert a needle into your testes and withdraw some sperm." C) "With proper stimulation of your genitals, you may be able to produce an erection and ejaculation." D) "It takes sympathetic innervation to produce an erection and ejaculation which is loss with your type of injury."

C) "With proper stimulation of your genitals, you may be able to produce an erection and ejaculation." Genital stimulation can produce erection and ejaculation in some men with complete transection of the spinal cord; therefore, that negates answer choices A and D.

Which of the following teaching points by an oncologist is most appropriate for a 33-year-old male who will begin treatment shortly for his testicular cancer? A) "Thanks to new treatment advances, there's a very good chance that I won't have to remove a testicle." B) "If steroid treatment fails, then we can consider surgery and/or chemotherapy." C) "You should know that there might be adverse effects on your sexual function after treatment is finished." D) "The advantage of performing an orchiectomy over other treatments is that it rules out recurrence."

C) "You should know that there might be adverse effects on your sexual function after treatment is finished." Orchiectomy is the standard treatment for testicular cancer. Steroidal treatments are not used, and sexual function is commonly affected. Recurrence is possible even after orchiectomy.

Following ejaculation in a healthy adult, approximately 300 million sperm is present. Which percent of these sperm would be considered normal (mobile and morphologically healthy) and able to fertilize an egg? A) 95% B) 75% C) 55% D) 45%

C) 55% Approximately 3 mL of semen is expressed with each ejaculate, and each milliliter is made up of about 100 million sperm. However, approximately 20% of the sperm in any ejaculate are not morphologically normal and about 25% are immobile.

The mother of 6-year-old male and female fraternal twins has brought her son to see a pediatrician because he is nearly 4 inches shorter than his sister. Which of the following phenomena would the physician most likely suspect as contributing factor to the boy's short stature? A) Genetic short stature B) Lack of IGF receptors in epiphyseal long bones C) A shortage of hypothalamic GHRH production D) Excess insulin production resulting in chronically low blood glucose levels

C) A shortage of hypothalamic GHRH production Inadequate levels of hypothalamic GHRH will result in adequate production but inadequate release of GH by the pituitary. Genetic short stature is less likely given the disparity between his height and his twin's, and a shortage of IGF receptors is not a noted pathology. While poorly controlled diabetes can contribute to short stature, excess insulin production is not a likely factor.

Which of the following hormones are derivatives of cholesterol? A) Epinephrine and norepinephrine B) Insulin and glucagon C) Aldosterone and testosterone D) Eicosanoids and retinoids

C) Aldosterone and testosterone Steroids such as aldosterone and testosterone are a classification of hormones that are derived from cholesterol. Epinephrine and norepinephrine are both amino acids, while insulin and glucagon are classified among peptides, polypeptides, proteins, and glycoproteins. Eicosanoids and retinoids consist of fatty acid compounds.

The Women's Health Initiative results have led the Institute of Medicine (2010) to recommend which of the following guidelines related to hip fractures in women? A) All postmenopausal women should take 1200 mg of calcium per day to prevent osteoporosis. B) The standard daily recommended amount of vitamin D was increased to 400 IU/day. C) All adults should take 600 IU/day of vitamin D to maintain healthy bones. D) A combination of calcium and vitamin D does not appear to prevent hip fractures.

C) All adults should take 600 IU/day of vitamin D to maintain healthy bones. The release of data from two clinical trials within the WHI (low-fat dietary patterns, calcium and vitamin D supplementation) has challenged conventional wisdom in other areas. The use of calcium (1000 mg/day) plus vitamin D (200 IU/day) was shown to result in a small but significant improvement in hip bone density but failed to reduce the risk of hip fractures. The IOM recommends that all adults should take 600 IU/day of vitamin D to maintain healthy bones.

A 71-year-old man has visited his family physician for a checkup, during which the physician has initiated a discussion about the client's sexual function. Which of the following phenomena would the physician most likely consider pathological rather than age-related changes? A) The presence of an enlarging prostate gland B) A decrease in the size and firmness of the client's testes C) Cessation of androgen production D) A decrease in the force of the man's ejaculation

C) Cessation of androgen production Absence of androgen production and release would be considered a pathological finding rather than a normal accompaniment of the aging process. Androgen replacement in aging men with low androgen levels shows an increase in lean body mass and a decrease in bone turnover. Although they would not necessarily remain untreated, hypogonadism, testicular atrophy, and a decreased force of ejaculation are common reproductive changes that are associated with advanced age.

A 38-year-old woman takes clomiphene, an infertility drug that works by competing with, and thereby blocking, cellular receptors for estrogen. Which of the following statements is most likely to be true of this client? A) Receptors for all other steroid hormones will also be blocked. B) Up-regulation will increase the number of estrogen receptors on each target cell. C) Estrogen will continue to pass freely through the cellular membranes. D) Laboratory tests will reveal an increase in cyclic adenosine monophosphate (cAMP) levels.

C) Estrogen will continue to pass freely through the cellular membranes. Because estrogen is a steroid hormone, its receptors in target cells are located inside the cell membrane, and their blockage does not affect the movement of the hormone into and out of the cell. Receptors are specific for each hormone, so no hormones other than estrogen will be blocked. Up-regulation occurs when hormone levels are decreased, and in this case, the estrogen level will increase. Second messengers, such as cAMP, are only activated by peptide hormones and catecholamines.

A pregnant client who is taking Risperidone (Risperdal), an antipsychotic prescribed for her bipolar disorder, is at high risk for developing which of the following hormonal adverse effects? A) Increased growth hormone production B) Decreased human chorionic gonadotropin levels C) Excess prolactin secretion D) Decreased follicle-stimulating hormone secretion

C) Excess prolactin secretion Although prolactin does not appear to play a physiologic role in ovarian function, hyperprolactinemia leads to hypogonadism. Hyperprolactinemia may occur as an adverse effect of drug treatment using phenothiazine derivatives (i.e., antipsychotic drugs that block dopamine receptors).

Having heard positive reports of the benefits of hormone therapy (HT) from her sister-in-law and friends, a 49-year-old woman has presented to her family physician asking to start HT. Her uterus is intact, and previous bone scans have indicated low bone density. The client also has a family history of heart disease. She characterizes her symptoms of menopause as "noticeable, but not debilitating by any means." Based on the most current research, what is her physician's best course of action? A) Begin estrogen-progesterone HT (EPT) to prevent future menopausal symptoms and coronary heart disease (CHD). B) Forego HT in light of her preexisting low bone density and consequent risk of osteoporosis. C) Forego HT but consider alternative therapies and reevaluate if her symptoms significantly affect her quality of life. D) Begin low-dose HT but perform regular breast cancer screening and heart health checks.

C) Forego HT but consider alternative therapies and reevaluate if her symptoms significantly affect her quality of life. Current recommendations for HT, in light of the findings of the WHI and other clinical trials, are to avoid HT for primary or secondary prevention of CHD; develop an individual risk profile for every woman contemplating HT and provide information regarding known risks; utilize HT only in those women who require relief from menopausal symptoms that affect quality of life; consider lower than standard doses and alternative routes of administration; limit the use of HT to the shortest duration consistent with goals, benefits, and risks of treatment for each woman; and because of the potential risks associated with HT products that are FDA approved for the prevention of postmenopausal osteoporosis, consider alternative therapies if the woman is not symptomatic.

A 51-year-old man is receiving his annual physical exam, and his care provider is explaining the rationale for performing a digital rectal exam (DRE). Which of the following statements best captures the rationale for the procedure? A) DRE, combined with a measurement of prostate-specific antigen (PSA), is the easiest way to confirm or rule out benign prostatic hyperplasia (BPH). B) The presence of an enlarged prostate provides a definitive diagnosis of prostate cancer. C) If the prostate is hardened on examination, a biopsy is indicated for further investigation. D) DRE is a screening test recommended for men who are experiencing either reduced urine flow or pain on urination.

C) If the prostate is hardened on examination, a biopsy is indicated for further Abnormalities detected during a DRE provide a rationale for further investigation by biopsy. PSA is used to screen for prostate cancer; however, it is also positive in BPH. The anatomic location of the prostate at the bladder neck contributes to the pathophysiology and symptomatology of BPH. A BPH is not a definitive sign of prostate cancer. DRE is recommended annually for all men over age 50.

A 54-year-old diabetic patient has come to the urology clinic complaining of erectile dysfunction. His history includes obesity, coronary artery disease that required CABG 3 years ago, hypertension, and gout. The nurse practitioner is reviewing his record in order to prescribe medication. The practitioner is considering prescribing sildenafil (Viagra). Which of the following home medications is contraindicated if taken concurrently with sildenafil? A) Diltiazem (Cardizem), a calcium channel blocker B) Cordarone (amiodarone), an antiarrhythmic C) Imdur (isosorbide mononitrate), a vasodilator D) Lasix (furosemide), a diuretic

C) Imdur (isosorbide mononitrate), a vasodilator Sildenafil (Viagra) is a selective inhibitor of phosphodiesterase type 5 (PDE-5), the enzyme that inactivates cGMP. This acts by facilitating corporeal smooth muscle relaxation in response to sexual stimulation. The concomitant use of PDE-5 inhibitors and nitrates (Imdur) is absolutely contraindicated because of the risk of profound hypotension. The other medications listed are not nitrates and do not have this adverse reaction.

A 51-year-old woman has been experiencing signs and symptoms of perimenopause and has sought help from her family physician. A deficiency in estrogen levels has been determined to be a contributing factor. Which of the following phenomena could potentially underlie the woman's health problem? A) Sufficient synthesis of estrogen but inadequate vesicle-mediated release B) Inadequate synthesis in the rough endoplasmic reticulum of her ovarian cells C) Insufficient estrogen production within the smooth endoplasmic reticulum of the relevant cells D) A lack of prohormone precursors needed for estrogen synthesis and release

C) Insufficient estrogen production within the smooth endoplasmic reticulum of the relevant cells Steroids such as estrogen are produced in the smooth endoplasmic reticulum. Synthesis and release are not separate processes as in the case of peptide hormones, and prohormones are associated with peptide, polypeptide, and protein hormones.

A 23-year-old woman has been referred to a fertility clinic after 1 year of attempting to become pregnant. Her diagnostic workup has resulted in a diagnosis of polycystic ovary syndrome (PCOS). What will the first line of treatment most likely consist of? A) Estrogen-progesterone hormone therapy and the administration of clomiphene B) Surgical resection of the ovaries using laparoscopy C) Lifestyle modifications to include weight loss by lowering calories and fat consumption D) Temporary use of oral contractive agents

C) Lifestyle modifications to include weight loss by lowering calories and fat consumption Lifestyle modifications are the treatment of choice for PCOS. Hormone therapy, surgery, and use of oral contraceptives are less likely to be a primary treatment option.

A 15-year-old female has presented to her family physician complaining of frequent discomfort around the time of her period. She has subsequently been diagnosed with primary dysmenorrhea. Which of the following treatments is most likely to be effective? A) Investigation and resolution of her hypothalamic-pituitary-ovarian disorder B) Hormone therapy aimed at resolving her estrogen deprivation C) Pain control with prostaglandin synthetase inhibitors D) Dilation and curettage

C) Pain control with prostaglandin synthetase inhibitors Because she has been diagnosed with primary rather than secondary dysmenorrhea, treatment of a specific underlying condition is not indicated. Treatment will likely focus on adequate pain control. D&C is not a relevant intervention.

A 51-year-old woman has been diagnosed with Cushing syndrome after a diagnostic workup that reveals cortisol hypersecretion. The nurse knows which of the following assessment findings would be inconsistent with her diagnosis? A) Increased blood pressure and decreased potassium levels B) A protruding abdomen and a "buffalo hump" on the back C) Poor stress management and hyperpigmentation D) A "moon face" and muscle weakness

C) Poor stress management and hyperpigmentation A low tolerance for stress and hyperpigmentation is associated with Addison disease and its consequent elevated levels of ACTH. High blood pressure, hypokalemia, buffalo hump, and moon face are all characteristics of the elevated steroid levels that denote Cushing syndrome.

A 21-year-old female is suspected of having inadequate function of her hypothalamic-pituitary-thyroid system. Her care provider is planning to inject thyrotropin-releasing hormone (TRH) and then measure her levels of TSH. Which of the following diagnostic tests is being performed? A) Suppression test B) Radioimmunoassay (RIA) test C) Stimulation test D) Metabolite excretion test

C) Stimulation test A stimulation test involves the introduction of an element that stimulates the production of another factor or hormone followed by measurement of that hormone. This is not the case in a suppression test, RIA test, or metabolite excretion test.

A couple who have three daughters would like to add a son to their family, and they have recently learned that the woman is pregnant. Which of the following phenomena would most likely be associated with the woman's eventual delivery of a healthy son? A) The production of dihydrotestosterone (DHT) by the embryo and DHT's differentiation into testosterone B) Development of testes under the influence of the X chromosome C) Suppression of female reproductive structures by anti-Müllerian hormone (AMH) D) Atrophy of the Wolffian ducts during embryonic development

C) Suppression of female reproductive structures by anti-Müllerian hormone (AMH) AMH suppresses the development of female genital structures and is a normal aspect of normal sex differentiation. Testosterone is a precursor to DHT rather than vice versa, and testes develop under the influence of the Y chromosome. The Wolffian ducts are a component of male sex differentiation.

A 14-year-old boy has been brought to the emergency department by his mother in excruciating pain that is radiating from his scrotum to his inguinal area. The boy's heart rate is 122 beats/minute, and he has vomited twice before arrival at the hospital. Examination reveals that his scrotum is reddened and slightly swollen, and the testes are firm to touch and tender, with extensive cremaster muscle contraction noted. What is the boy's most likely diagnosis? A) Epididymitis B) Hydrocele C) Testicular torsion D) Varicocele

C) Testicular torsion The combination of the boy's age, signs, and symptoms is indicative of testicular torsion. Epididymitis normally lacks cremaster muscle involvement, and hydrocele is marked by massive distention of the scrotum. Varicocele is often asymptomatic or marked by heaviness in the scrotum.

Which of the following statements most accurately captures a characteristic of the external female genitalia? A) Skene and Bartholin glands perform endocrine and exocrine functions. B) Surgical repair is required in the case of a ruptured hymen in prepubescent females. C) The labia majora and clitoris are analogous to the male scrotum and penis, respectively. D) The external genitalia facilitate sexual function and hormonal regulation.

C) The labia majora and clitoris are analogous to the male scrotum and penis, respectively. Embryonic differentiation results in an anatomical relationship between the labia majora and the scrotum and between the clitoris and the penis. There is no identified hormonal/endocrine role of the structures of the external genitalia, and a ruptured hymen does not necessitate medical intervention.

Which of the following statements best captures an aspect of the neural control of sexual function? A) The sensory system contained in the testes communicates with the spinal cord by way of afferent pathways. B) Perineal stimulation transmits signals to the thalamus and hypothalamus. C) The spinal cord plays a central role, with the limbic system and cerebral cortex also contributing. D) Sympathetic stimulation causes the shunting of blood resulting in erection, while parasympathetic stimulation results in detumescence.

C) The spinal cord plays a central role, with the limbic system and cerebral cortex also contributing. Afferent impulses from sensory receptors in the glans penis pass through the pudendal nerve to ascending fibers in the spinal cord by way of the sacral plexus, while stimulation of other perineal areas can transmit signals to higher brain centers, such as the limbic system and cerebral cortex. The thalamus and hypothalamus do not play a central role in the neural control of male sexual function, and arousal can result from parasympathetic stimulation.

A 71-year-old man diagnosed with a stage T2 prostate tumor 2 years ago has elected watchful waiting, based upon an underlying heart condition that renders surgery potentially fatal. Recently, his PSA has jumped considerably, as have his levels of serum acid phosphatase. Which course of action would be least appropriate at this stage? A) Combination treatment with an antiandrogen and a GnRH agonist B) Treatment with bisphosphonates C) Treatment with GnRH agonists alone D) Treatment with ketoconazole

C) Treatment with GnRH agonists alone The sharp rise in PSA, coupled with an increase in levels of serum acid phosphatase, is strongly indicative of metastatic cancer (which could be confirmed via molecular imaging such as MRI). Unopposed GnRH agonists initially cause LH and FSH levels to rise, stimulating the production of testosterone, which acts as fuel for the fire of prostatic metastasis. Thus, their use alone would not be appropriate. However, if these agonists are combined with antiandrogens, testosterone levels can be quelled from two different fronts simultaneously. Ketoconazole is a chemical castrating agent that could bring down testosterone levels rapidly and might be more appropriate for cases of advanced and widespread metastasis. Bisphosphonates address the effects of metastatic bone involvement and of osteoporosis resulting from antiandrogen therapy.

Which of the following statements best captures an aspect of the role of progesterone? Progesterone A) production begins shortly after conception and peaks prior to delivery. B) is responsible for the stimulation of lactation in the postpartum period. C) levels peak after ovulation and is present throughout the menstrual cycle. D) is the primary hormone responsible for the development of female secondary sex characteristics.

C) levels peak after ovulation and is present throughout the menstrual cycle. Progesterone is present in varying levels at all times in females, but peaks following ovulation. It is not responsible for lactation and does not have primary responsibility for the development of secondary sex characteristics.

An example of a single hormone that can exert effects in different tissues, erythropoietin, made in the kidney stimulates the bone marrow to produce A) platelets. B) natural killer cells. C) red blood cells. D) mast cells.

C) red blood cells. A characteristic of hormones is that a single hormone can exert various effects in different tissues. For example, erythropoietin, a traditional circulating hormone, is made in the kidney and stimulates erythropoiesis in the bone marrow.

Since steroid hormones are bound to protein carriers for transport, this means A) they are water soluble and circulate freely in the blood. B) they are degraded by enzymes in the blood. C) they are inactive in the bound state. D) they will be converted into a useable form by enzymes in the blood.

C) they are inactive in the bound state. Steroid hormones are bound to protein carriers for transport and are inactive in the bound state. Their activity depends on the availability of transport carriers.

Staff at a women's health center are being briefed by their supervisor on the latest recommendations for breast cancer screening. Which of the following guidelines should staff pass on to their clients? A) "Breast self-exam should be ideally performed around the time of the month that you think you're ovulating." B) "The most important thing you can do to identify breast cancer early is to perform regular, systematic self-exams." C) "All postmenarche females should get a clinical exam by a trained professional on an annual basis." D) "Screening mammography and clinical exams are the cornerstones of breast cancer screening."

D) "Screening mammography and clinical exams are the cornerstones of breast cancer screening." Self-exam has recently been deemphasized by the American Cancer Society with emphasis now placed on mammography and trained exam. If done, self-exam should be performed around the time of menses, and clinical exams every 3 years are believed to be sufficient for women under 40.

An endocrinologist is providing care for a 30-year-old male who has lived with the effects of increased levels of growth hormone (GH). Which of the following teaching points about the client's future health risks is most accurate? A) "It's not unusual for high GH levels to cause damage to your hypothalamus." B) "GH excess inhibits your pancreas from producing enough insulin." C) "The high levels of GH that circulate in your body can result in damage to your liver." D) "When your pituitary gland is enlarged, there's a real risk that you'll develop some sight deficiencies."

D) "When your pituitary gland is enlarged, there's a real risk that you'll develop some sight deficiencies." GH excess is associated with tumor formation and consequent compression of cranial nerves responsible for vision. Damage to the hypothalamus and liver is not common sequelae. While the beta cells of the pancreas can "burn out," the primary effect of excess GH is to increase insulin secretion.

A care aide at a long-term care facility has left a note for the care director stating that an 82-year-old resident has a grossly distended scrotum and a likely inguinal hernia. On examination, the resident has been diagnosed instead with hydrocele. What will the care team most likely tell the resident and his family about his diagnosis? A) "We'll get you to the hospital quickly, because if this isn't treated, it can result in a blockage in the blood flow to your testes." B) "This isn't a result of your intestine entering your scrotum, but an accumulation of fluid within your testes." C) "This can sometimes result from the lower level of sex hormones that comes with age, so you will likely benefit from a testosterone supplement." D) "While distressing to look at, this condition usually doesn't have any significant consequences and won't need treatment."

D) "While distressing to look at, this condition usually doesn't have any significant consequences and won't need treatment." Hydrocele in adult males is normally considered benign condition that does not warrant treatment unless mobility is affected. Fluid accumulates in the space between the tunica vaginalis and tunica albuginea, not within the testes themselves, and hormone therapy is not indicated.

A student asks the instructor what are the parameters of infertility. The instructor responds, "Infertility is defined as" A) 100 million of sperm in ejaculate. B) 50% of sperm has good motility. C) morphology that appears normal. D) 10 million/mL sperm available in seminal fluid.

D) 10 million/mL sperm available in seminal fluid. The sperm count in a normal ejaculate is approximately 100 to 400 million. Infertility may occur when insufficient numbers of motile, healthy sperm are present. A "fertile sample" on seminal fluid analysis is associated with a count greater than 20 million/mL, greater than 50% motility, normal morphology, and a volume of 1.5 to 6 mL.

When explaining about structural classifications to a group of students, the instructor discusses the peptides and proteins. They talk about small hormones and hormones as large and complex as growth hormone (GH), which has approximately how many amino acids involved? A) 50 amino acids B) 100 amino acids C) 150 amino acids D) 200 amino acids

D) 200 amino acids Growth hormone is a very large and complex protein that has approximately 200 amino acids.

Which of the following women is most likely to have a sexually transmitted infection as a contributing factor to her health problem? A) A 29-year-old woman with a diagnosis of localized vulvodynia B) A 40-year-old who is being treated for vaginal cancer C) A 32-year-old who is undergoing diagnostics to rule out endometriosis D) A 41-year-old with a diagnosis of mucopurulent cervicitis

D) A 41-year-old with a diagnosis of mucopurulent cervicitis While C. albicans, T. vaginalis, Neisseria gonorrhoeae, Gardnerella vaginalis,Chlamydia trachomatis, Ureaplasma urealyticum, and herpes simplex virus can all contribute to cervicitis. C. trachomatis is the organism most commonly associated with mucopurulent cervicitis. Vulvodynia, vaginal cancer, and endometriosis are less likely to have a sexually transmitted etiology.

After receiving change-of-shift report about the following four patients, which patient should the nurse assess first? A) A 22-year-old admitted with SIADH who has a serum sodium level of 130 mEq/L B) A 31-year-old who has iatrogenic Cushing syndrome with a capillary blood glucose level of 204 mg/dL C) A 53-year-old who has Addison disease and is due for a scheduled dose of hydrocortisone (Solu-Cortef) D) A 70-year-old returning from PACU following partial thyroidectomy who is extremely agitated, has an irregular pulse rate of 134, and has an elevated temperature of 103.2°F

D) A 70-year-old returning from PACU following partial thyroidectomy who is extremely agitated, has an irregular pulse rate of 134, and has an elevated temperature of 103.2°F Manipulation of a hyperactive thyroid gland during thyroidectomy can cause thyroid storm. It is manifested by very high fever, extreme cardiovascular effects (tachycardia, HF, angina), and severe CNS effects (agitation, restlessness, and delirium). Answer choice A refers to normal sodium levels. Answer choice B refers to high blood glucose level but not critical level. Answer choice C refers to lower priority. It is always preferred to give medications in a timely manner; however, thyroid storm signs and symptoms are the priority for this group of patients.

A 28-year-old female has been told she has atypical glandular cells following Pap smear. The physician will likely recommend which procedure that can be performed in his office to remove the abnormal zone and provide a specimen for further histological evaluation? A) Cone biopsy B) Intracavitary irradiation C) Trachelectomy D) Loop electrosurgical excision procedure (LEEP)

D) Loop electrosurgical excision procedure (LEEP) Because adenocarcinoma of the cervix is being detected more frequently, especially in women younger than 35 years of age, a Pap smear result of atypical glandular cells warrants further evaluation. The LEEP has taken the place of cone biopsies in most situations and is now the first-line management for CIN II/III. This outpatient procedure allows for the simultaneous diagnosis and treatment of dysplastic lesions found on colposcopy. In skilled hands, this wire can remove the entire transformation zone, providing adequate treatment for the lesion while obtaining a specimen for further histological evaluation. Cone biopsy is a surgical procedure that removes a cone-shaped wedge of cervix. Trachelectomy is removal of the cervix, and intracavitary irradiation is a form of brachytherapy to treat cervical cancer.

A patient is admitted to the hospital in adrenal crisis 1 month after a diagnosis of Addison disease. The nurse knows which of the following clinical manifestations would support this diagnosis? A) Hyperactive deep tendon reflexes and slow, shallow breathing B) Cerebral spinal fluid leakage and impaired swallowing C) Irregular heart rate and decreased temperature D) Change in the level of consciousness and profound hypotension

D) Change in the level of consciousness and profound hypotension Acute adrenal crisis is a life-threatening situation. Exposure to even a minor illness or stress can cause a client with Addison disease to develop nausea, vomiting, muscular weakness, hypotension, dehydration, and vascular collapse (which causes a change in LOC). Hemorrhage (low BP) can be caused by septicemia, adrenal trauma, anticoagulant therapy, adrenal vein thrombosis, or adrenal metastases. A hyperactive reflex may indicate disease of the pyramidal tract above the level of the reflex arc being tested. Generalized hyperactivity of DTRs may be caused by hyperthyroidism. Any tear or hole in the membrane that surrounds the brain and spinal cord (dura) can allow the fluid that surrounds those organs to leak. This fluid is called the cerebrospinal fluid (CSF). When it leaks out, the pressure around the brain and spinal cord drops. Causes of leakage through the dura include certain head, brain, or spinal surgeries; head injury; placement of tubes for epidural anesthesia or pain medications; or lumbar puncture. Irregular heart rates (arrhythmias) may be caused by many different factors, including coronary artery disease; electrolyte imbalances in your blood (such as sodium or potassium); changes in your heart muscle; or injury from a heart attack.

Events in the ovulatory cycle progress in a similar manner each month. Place the following events in the ovulatory cycle in the correct chronological order. Use all the options. A) Formation of the theca B) Formation of the corpus luteum C) Formation of the antral follicle D) Development of the zona pellucida E) Bursting of the mature follicle

D) Development of the zona pellucida A) Formation of the theca C) Formation of the antral follicle E) Bursting of the mature follicle B) Formation of the corpus luteum The zona pellucida surrounds a primary oocyte as it transitions to a secondary follicle, after which the theca surrounds the follicle. The antrum is subsequently formed around the maturing follicle, and it eventually bursts. After ovulation, the corpus luteum is established marking the beginning of the luteal stage of the ovulatory cycle.

Which of the following statements best captures an aspect of normal spermatogenesis? A) Testosterone chemically lyses each primary spermatocyte into two secondary spermatocytes with 23 chromosomes each. B) Sertoli cells differentiate into spermatids, each of which can contribute half of the chromosomes necessary for reproduction. C) Spermatogonia adjacent to the tubular wall undergo meiotic division and provide a continuous source of new germinal cells. D) Each primary spermatocyte undergoes two nuclear divisions yielding four cells with 23 chromosomes each.

D) Each primary spermatocyte undergoes two nuclear divisions yielding four cells with 23 chromosomes each. The process of meiosis consists of two consecutive nuclear divisions of a primary spermatocyte with formation of four daughter cells, each containing a single set of 23 chromosomes. Testosterone does not chemically lyse immature sperm, and Sertoli cells play a nurturing and facilitative role in spermatogenesis rather than differentiating themselves into spermatocytes. Spermatogonia undergo mitosis, not meiosis.

Following the identification of low levels of T3 and T4 coupled with the presence of a goiter, a 28-year-old female has been diagnosed with Hashimoto thyroiditis. In light of this diagnosis, which of the following assessment results would constitute an unexpected finding? A) The presence of myxedema in the woman's face and extremities B) Recent weight gain despite a loss of appetite and chronic fatigue C) Coarse, dry skin and hair with decreased sweat production D) Increased white cell count and audible crackles on chest auscultation

D) Increased white cell count and audible crackles on chest auscultation An increased white cell count and the presence of adventitious fluid in the lungs are not classic findings associated with hypothyroidism. Myxedema, weight gain, lethargy, and dry skin and nails are commonly associated with low levels of thyroid hormones.

A 39-year-old male client has been recently diagnosed with primary hypogonadism. Which of the following lab results would be most indicative of this diagnosis? A) Normal levels of free testosterone; low levels of total testosterone B) Low free testosterone, luteinizing hormone (LH), and follicle-stimulating hormone (FSH) levels C) Low levels of gonadotropin-releasing hormone (GnRH) D) Low testosterone levels; high levels of LH and FSH

D) Low testosterone levels; high levels of LH and FSH Primary hypogonadism is associated with a defect in the testicular production of testosterone normally accompanied by high levels of the hormonal precursors. Free testosterone levels would not be normal given the client's diagnosis. Low GnRH levels are associated with tertiary hypogonadism.

Which of the following statements best captures the relationship between the hypothalamus and the pituitary gland as it relates to endocrine function? A) The hypothalamus directly measures the levels of most hormones throughout the body and inhibits or stimulates the pituitary accordingly. B) The pituitary gland coordinates and dictates the release of hormones from the hypothalamus that act on their intended target cells. C) The pituitary gland and hypothalamus have two-way communication that mediates the signals from neuronal inputs. D) The hypothalamus receives input from numerous sources throughout the body and directs the pituitary to then control many target glands and cells.

D) The hypothalamus receives input from numerous sources throughout the body and directs the pituitary to then control many target glands and cells. The hypothalamus can be viewed as a bridge by which signals from multiple systems are relayed to the pituitary gland. The hypothalamus collects data from sources throughout the body rather than directly measuring levels, and communication normally flows from the hypothalamus to the pituitary.

A 40-year-old woman has been identified to have a deficiency in estrogen. Which of the following physiological phenomena is most likely to remain unaffected? A) Parathyroid hormone antagonism and the rate of bone resorption B) The regulation of uterine endothelial development C) The maintenance of normal skin and blood vessel structure D) The synthesis and release of adrenal glucocorticoids

D) The synthesis and release of adrenal glucocorticoids Synthesis and release of adrenal glucocorticoids are not directly influenced by estrogen, unlike the processes of bone resorption, endothelial regulation, and the maintenance of skin and blood vessel structure.

Which of the following situations would be considered pathological in an otherwise healthy, 30-year-old female? A) The woman's ovaries are not producing new ova. B) The woman's ovaries do not synthesize or secrete luteinizing hormone (LH). C) The epithelium covering the woman's ovaries is broken during the time of ovulation. D) The woman's ovaries are not producing progesterone.

D) The woman's ovaries are not producing progesterone. As the number of ova within an ovary is fixed at birth and decreases over time, a lack of production of ova is normal. The epithelial lining is normally broken during ovulation, and the LH is produced by the pituitary gland. A lack of progesterone would be considered pathological.

A client with a history of an endocrine disorder exhibits signs and symptoms of hormone deficiency. Which of the following processes would the client's care team most likely rule out first as a contributing factor? A) The client's target cells lack sufficient receptors for the hormone in question. B) Hormone production is sufficient, but affinity on the part of the target cells is lacking. C) The process of down-regulation has resulted in decreased hormone sensitivity. D) Up-regulation has increased the sensitivity of the body to particular hormone levels.

D) Up-regulation has increased the sensitivity of the body to particular hormone levels. Up-regulation is a response to low hormone levels in which the number of receptors increases. As such, it would not likely result in signs and symptoms of deficiency but is rather a compensatory mechanism that counters a deficiency. Insufficient numbers of receptors, low affinity, and down-regulation could all contribute to signs and symptoms of a hormone deficiency.

A college student has come to the health clinic complaining of heavy bleeding during and between her menstrual periods. The nurse practitioner will document this as A) dyspareunia. B) amenorrhea. C) polymenorrhea. D) menometrorrhagia.

D) menometrorrhagia. Menometrorrhagia is heavy bleeding during and between menstrual periods. Polymenorrhea is frequent menstruation with periods less than 21 days apart. Amenorrhea is absence of menstruation. Dyspareunia is pain following intercourse.

Growth hormone (GH) secretion is inhibited by A) hypoglycemia. B) starvation. C) heavy exercise. D) obesity.

D) obesity. GH is inhibited by increased glucose levels, free fatty acid release, cortisol, and obesity. It is stimulated by hypoglycemia, fasting starvation, increased blood levels of amino acids, and stress conditions such as trauma, excitement, emotional stress, and heavy exercise.

A woman has gone to her gynecologist complaining of pain during intercourse. The physician orders a maturation index on her vaginal scrapings. The results return an index of 75-25-0. The nurse can interpret this to mean A) she is in the beginning of cervical cancer. B) she is in the prime of her reproductive years. C) she is in a perimenopausal phase. D) she is experiencing postmenopausal vaginal dryness.

D) she is experiencing postmenopausal vaginal dryness. The vaginal scrapings are used for a test, the maturation index, that examines the cellular structure and configuration of the vaginal epithelial cells. Typically, this index is 0-40-60 during the reproductive years. With diminished estrogen levels, there is a shift to the left, producing an index of 30-40-30 during the perimenopausal period, and an index of 75-25-0 occurs during the postmenopausal period.

Sperm follow a prescribed route through the excretory ducts of the male reproductive system. Place the following components of the genital duct system in the order that sperm follow. Use all the options. A) Efferent ductules B) Ductus deferens C) Prostatic urethra D) Urethra E) Seminiferous tubules F) Epididymis

E) Seminiferous tubules A) Efferent ductules F) Epididymis B) Ductus deferens C) Prostatic urethra D) Urethra Seminiferous tubules combine in the testes to form the rete testes, which flow into the efferent ductules and the epididymis. The epididymis flows into the ductus deferens, which leaves the testes passing through the prostatic urethra before leaving the body by way of the urethra.


Related study sets

Anatomy and Physiology Chapter One

View Set

Autonomic Nervous System and Central Nervous System

View Set

Financial Management Midterm Review

View Set

Chapter 7: Improving Decisions with Marketing Information

View Set

Henry David Thoreau - "Civil Disobedience"

View Set